4
$\begingroup$

I would like to ask the following question. This is related to one lemma that I need in a recent research on the arithmetic behavior of transcendental functions with integer coefficients.

Let $\alpha, \beta\in (-1,1)$, with $\alpha\neq \pm \beta$. I would like to prove that there exists a function $f\in \mathbb{Z}[[z]]$ analytic in the unit ball and such that $f(\beta)=0$ and $f(\alpha)\neq 0$.

I am able to prove this when $\alpha\cdot \beta<0$ and when $|\beta|>|\alpha|$.

Any help?

$\endgroup$
8
  • 1
    $\begingroup$ How did you prove it for the cases you mentioned? $\endgroup$
    – user78249
    Nov 17, 2016 at 22:57
  • 5
    $\begingroup$ Is this a homework question? $\endgroup$ Nov 17, 2016 at 23:06
  • 1
    $\begingroup$ it could be, but it appears in order to prove one lemma in one of my recent works. $\endgroup$
    – Diego
    Nov 17, 2016 at 23:14
  • 2
    $\begingroup$ @PietroMajer When an anonymous asks to prove a homework-level "lemma", the burden is not us to prove that it is a student trying to have their homeworks solved. $\endgroup$
    – Boris Bukh
    Nov 18, 2016 at 2:17
  • 6
    $\begingroup$ @BorisBukh Of course the burden is not to us. On the other hand, if we raise a mistrust of somebody's purposes, it is generally acknowledged that we should at least provide a more solid reason than just smell. That's why (IMHO) it is better to stay on the question, objecting either its content or form, not the questioner's good faith. If a question appears too elementary to me, I kindly suggest a more suitable site. But note, what is trivial to you may be difficult to other professional mathematicians, just because it is not their field, and this is a situation where MO proves greatly useful. $\endgroup$ Nov 18, 2016 at 10:06

1 Answer 1

16
$\begingroup$

We may assume w.l.o.g $0<\beta<1$. Write $\beta^{-1}$ in the $ \beta^{-2}$ expansion as: $$\beta^{-1}=\sum_{k=0}^\infty d_k\beta^{2k}$$ with integer digits $0\le d_k<\beta^{-2}$, and define $$f(x):=-1+\sum_{k=0}^\infty d_kx^{2k+1}$$ Then $f\in\mathbb{Z}[[x]]$ is analytic in the unit disk, strictly increasing on the interval $(-1,1)$, and $f(\beta)=0$.

$\endgroup$
6
  • 1
    $\begingroup$ @Diego $f$ is strictly increasing, so it has only one zero. $\endgroup$
    – Wojowu
    Nov 18, 2016 at 11:12
  • 1
    $\begingroup$ Thanks @Pietro, it works. Another question, how to manage when $\alpha$ and $\beta$ are complex numbers inside the unit ball with $\alpha$ different of $\beta$ and $\overline{\beta}$? $\endgroup$
    – Diego
    Nov 18, 2016 at 12:12
  • $\begingroup$ Good question! Making $f\in\mathbb{Z}[[x]]$ an univalent (slicht) map on the unit disk seems hard to me (but it should be known, and it is worth posting a new question). The sufficient condition on coefficients for univalence I know is Alexander's one: $n|a_n|$ be decreasing, but for $f\in\mathbb{Z}[[x]]$ it implies $f$ is a polynomial. On the other hand, de Branges' necessary condition $|a_n|\le n|a_1|$ leaves some hope. If univalence proves hard or impossible, one should try to build an $f$ depending on both $\alpha$ and $\beta$, not only on $\beta$, possibly not slicht. $\endgroup$ Nov 18, 2016 at 13:18
  • $\begingroup$ Thanks @PietroMajer. I already posed the other question. I think the better idea is to consider a construction depending on both, but I am afraid that I need to split such a construction in many cases depend on the their arguments (rational, irrational, linearly independent, etc). $\endgroup$
    – Diego
    Nov 18, 2016 at 20:42
  • $\begingroup$ If you are ok with $f$ depending on both $\alpha$ and $\beta$, then I think the above construction can be repeated. "Complex base system" can be considered like in the real case. I think, given a base $\beta$, a large enough set of coefficients $C$ allows to represent any complex number, so you can make plenty of $f\in\mathbb{Z}[[z]]$ with bounded coefficients such that $f(\beta)=0$; then also getting $f(\alpha)\neq0$ should not be difficult. $\endgroup$ Nov 18, 2016 at 21:39

Your Answer

By clicking “Post Your Answer”, you agree to our terms of service and acknowledge you have read our privacy policy.

Not the answer you're looking for? Browse other questions tagged or ask your own question.